FMGE Mock Test Questions And Answers

Reviewed by Editorial Team
The ProProfs editorial team is comprised of experienced subject matter experts. They've collectively created over 10,000 quizzes and lessons, serving over 100 million users. Our team includes in-house content moderators and subject matter experts, as well as a global network of rigorously trained contributors. All adhere to our comprehensive editorial guidelines, ensuring the delivery of high-quality content.
Learn about Our Editorial Process
| By Fmgs India
F
Fmgs India
Community Contributor
Quizzes Created: 15 | Total Attempts: 39,999
| Attempts: 19,354 | Questions: 200
Please wait...
Question 1 / 200
0 %
0/100
Score 0/100
1. Karyotyping is done for:

Explanation

Karyotyping is a genetic test that involves analyzing the chromosomes in a sample of cells. It is used to detect and diagnose chromosomal disorders, which are abnormalities or changes in the structure or number of chromosomes. This test can identify conditions such as Down syndrome, Turner syndrome, and Klinefelter syndrome. Karyotyping is not typically used for diagnosing autosomal recessive (AR) or autosomal dominant (AD) disorders, as these types of disorders are caused by specific gene mutations rather than chromosomal abnormalities. Therefore, the correct answer is "Chromosomal disorders."

Submit
Please wait...
About This Quiz
FMGE Mock Test Questions And Answers - Quiz

Are you preparing for a screening test for medical sciences? Have a look at this FMGE mock test questions and answers, and test your knowledge. This quiz is... see morea mock test for your preparation for the final exam. So, take this quiz and prepare perfectly for all the upcoming tests and exams. Even if you miss out on something, we are here to help you with the correct answers. All the best for the best scores on this quiz. You can share it with other aspirants also.
see less

2. Artery in anatomical snuff box:

Explanation

The correct answer is radial because the anatomical snuff box is a triangular depression on the radial aspect of the dorsum of the hand. It is formed by the tendons of the extensor pollicis longus and brevis muscles. The radial artery runs through the anatomical snuff box, making it the artery present in this anatomical structure.

Submit
3. The receptor of HIV:

Explanation

CD4 is the correct answer because it is a receptor found on the surface of certain immune cells, including T-helper cells, which are the primary target of the HIV virus. HIV binds to the CD4 receptor on these cells, allowing the virus to enter and infect them. CD8 is another type of immune cell receptor, but it is not the primary receptor for HIV. Plasma cells are a type of immune cell that produces antibodies, and CD56 is a marker found on natural killer cells, neither of which are the primary receptor for HIV.

Submit
4. Which of the following is protective against carcinoma colon?

Explanation

A high fiber diet is protective against carcinoma colon because it helps to regulate bowel movements and prevent constipation. It also helps to maintain a healthy weight, which is important in reducing the risk of colon cancer. Fiber-rich foods also contain antioxidants and phytochemicals that have been shown to have anti-cancer properties. Additionally, a high fiber diet can help to lower cholesterol levels and control blood sugar, which are factors that are associated with an increased risk of colon cancer.

Submit
5. Neurofibromatosis is associated with all except:

Explanation

Neurofibromatosis is a genetic disorder characterized by the growth of tumors on nerves throughout the body. It is inherited in an autosomal dominant pattern, meaning that a person only needs to inherit one copy of the mutated gene to develop the disorder. Therefore, it is not associated with autosomal recessive inheritance. Cutaneous fibromas, cataract, and scoliosis are all commonly associated with neurofibromatosis.

Submit
6. A person sits from standing position, change seen is:

Explanation

When a person sits from a standing position, there is an increase in venous return. This means that more blood is flowing back to the heart from the veins. This increase in venous return leads to an increase in preload, which is the amount of blood filling the heart before it contracts. As a result, the heart rate increases to pump out this increased volume of blood. The other options, such as increase in epinephrine and increased cerebral blood flow, may also occur during this transition, but the primary change seen is the increase in venous return.

Submit
7. A 34-year-old female presents with symptoms of cough, dyspnea, and lymphadenopathy. Physical exam shows cervical adenopathy and hepatomegaly. Her chest radiograph is shown below. How should you pursue diagnosis?

Explanation

Based on the given information, the patient is presenting with symptoms of cough, dyspnea, cervical adenopathy, and hepatomegaly. The chest radiograph is shown to further investigate the cause of these symptoms. Bronchoscopy and transbronchial lung biopsy would be the most appropriate next step in pursuing a diagnosis. This procedure allows for direct visualization and sampling of the lung tissue, which can help identify any abnormalities or underlying conditions causing the patient's symptoms. Liver biopsy would not be the most appropriate option as the symptoms and physical exam findings are not indicative of a primary liver problem. Open lung biopsy and scalene node biopsy are not the first-line procedures for investigating the cause of the patient's symptoms.

Submit
8. Sleep is primarily regulated by:

Explanation

The hypothalamus is responsible for regulating sleep primarily. It contains a group of nerve cells called the suprachiasmatic nucleus (SCN), which acts as the body's internal clock. The SCN receives information about light and darkness from the eyes and sends signals to other parts of the brain, including the pineal gland, to regulate the production of melatonin, a hormone that helps control sleep-wake cycles. Additionally, the hypothalamus plays a role in controlling other sleep-related functions, such as body temperature and hormone production.

Submit
9. After cutting DNA with restriction enzymes; segments are joined by:

Explanation

After cutting DNA with restriction enzymes, the segments are joined by ligase. Ligase is an enzyme that catalyzes the formation of phosphodiester bonds between the sugar-phosphate backbones of DNA strands. It plays a crucial role in DNA replication, repair, and recombination. In this context, after the restriction enzymes cut the DNA at specific recognition sites, ligase is responsible for sealing the gaps and joining the DNA segments back together. This process is essential for the proper functioning and stability of the DNA molecule.

Submit
10. Q fever is transmitted by:

Explanation

Q fever is caused by the bacteria Coxiella burnetii and is primarily transmitted to humans through ticks. Ticks are known to carry and transmit the bacteria to both animals and humans. When an infected tick bites a human, it can transmit the bacteria into the bloodstream, leading to Q fever infection. Other modes of transmission, such as mites, lice, and aerosol, are not commonly associated with Q fever. Therefore, ticks are the most likely source of transmission for Q fever.

Submit
11. Laryngeal pseudosulcus true is:

Explanation

Laryngeal pseudosulcus is a condition characterized by a linear groove or furrow on the posterior aspect of the vocal folds. It is commonly associated with reflux induced laryngitis, also known as pharyngoesophageal reflux. This occurs when stomach acid flows back into the throat, causing irritation and inflammation of the larynx. The acid damages the lining of the larynx, leading to the formation of the pseudosulcus. Vocal abuse, tuberculosis, and high dose corticosteroids are not typically associated with the development of laryngeal pseudosulcus.

Submit
12.  Gold standard to diagnosis insulinoma is:

Explanation

The gold standard for diagnosing insulinoma is 72 hours fasting blood glucose levels and insulin levels. This is because insulinoma is a rare tumor that causes excessive production of insulin, leading to low blood sugar levels. By fasting for 72 hours, the body's insulin production should decrease, causing blood glucose levels to rise. If insulin levels remain high despite fasting, it suggests the presence of insulinoma.

Submit
13. Staging for Wilms syndrome:

Explanation

The correct answer is "International staging international society of pediatrics (ISOP)". This suggests that the International Society of Pediatrics (ISOP) has developed a staging system specifically for Wilms syndrome. This staging system likely helps classify the extent and severity of the disease, allowing for better understanding and management of the condition. The other options mentioned (Chadwick, AJCC, TNM) do not seem to be relevant to the staging of Wilms syndrome.

Submit
14. Prunning of vessels is seen in:

Explanation

Pruning of vessels refers to the process of reducing or eliminating small, peripheral blood vessels in order to redirect blood flow to more vital areas. In the context of primary pulmonary hypertension, the condition is characterized by the narrowing and constriction of pulmonary arteries, leading to increased resistance in the lungs. This can result in the pruning of vessels as the body attempts to redirect blood flow to areas with less resistance. Therefore, primary pulmonary hypertension is the most likely condition in which pruning of vessels is observed.

Submit
15. Chief cells are found in:

Explanation

Chief cells are found in the fundus of the stomach. The fundus is the upper portion of the stomach, located above the body and near the esophagus. Chief cells are responsible for secreting pepsinogen, an inactive form of the enzyme pepsin, which helps in the digestion of proteins. The presence of chief cells in the fundus allows for the initiation of protein digestion in the stomach.

Submit
16. In HIV maximum risk of transmission is by:

Explanation

The maximum risk of transmission of HIV is through blood transfusion. This is because HIV can be present in the blood of an infected person, and if this blood is transfused into another person, there is a high likelihood of transmission. Other modes of transmission such as homosexual activity, heterosexual activity, and needle pricks also carry a risk of transmission, but the risk is comparatively lower than that of blood transfusion.

Submit
17. Characteristic of acute inflammation:

Explanation

Acute inflammation is characterized by vasodilation and increased vascular permeability. Vasodilation refers to the widening of blood vessels, which allows for increased blood flow to the affected area. This helps deliver immune cells and nutrients to the site of inflammation. Increased vascular permeability refers to the loosening of the blood vessel walls, allowing fluid, proteins, and immune cells to leak into the surrounding tissues. This leads to swelling and redness at the site of inflammation. Together, vasodilation and increased vascular permeability help facilitate the inflammatory response and aid in the healing process.

Submit
18. Hallmark feature of benign HTN is:

Explanation

The hallmark feature of benign hypertension is hyaline arteriosclerosis. Hyaline arteriosclerosis refers to the thickening and hardening of the walls of small arteries and arterioles due to the deposition of a protein called hyaline. This condition is commonly seen in individuals with long-standing hypertension and is characterized by the narrowing of blood vessels, reduced blood flow, and potential organ damage. Hyaline arteriosclerosis is considered a marker of chronic hypertensive vascular disease and is often associated with conditions such as diabetes and aging.

Submit
19. Which of the following is the recommended timing for the administration of the first dose of the Hepatitis B (HB) vaccine to a neonate?

Explanation

The first dose of the Hepatitis B vaccine is recommended to be administered within 12 hours of birth, especially for infants born to mothers who are Hepatitis B positive. This helps to prevent the transmission of the virus from mother to child. The other options are incorrect because they represent delayed administration of the vaccine, which increases the risk of infection.

Submit
20. An intrinsic factor of the castle is present in:

Explanation

Parietal cells are present in the castle and have an intrinsic factor. The intrinsic factor is a glycoprotein that is essential for the absorption of vitamin B12 in the small intestine. It is produced by the parietal cells of the stomach lining and binds to vitamin B12, allowing it to be absorbed by the body. This intrinsic factor is necessary for the proper functioning of the digestive system and the production of healthy red blood cells.

Submit
21. Extensive abrasions are found on the body of a pedestrian lying by the road side. What is the cause?

Explanation

The extensive abrasions found on the body of a pedestrian lying by the road side suggest that the cause of these injuries is a secondary impact injury. This means that the pedestrian was likely hit by a vehicle or another object after the initial impact, resulting in the abrasions on their body.

Submit
22. Amifostine is protective to all except:

Explanation

Amifostine is a drug that is used as a protective agent against the harmful effects of radiation and certain chemotherapy drugs. It works by scavenging and neutralizing harmful free radicals that are produced during these treatments. While amifostine provides protection to various organs and tissues, it does not have a significant protective effect on the central nervous system (CNS). Therefore, the correct answer is CNS.

Submit
23. A 7-year-old girl presents with bleeding in joints. She has prolonged aPTt, normal PT an platelet counts. What could be the deficiency?

Explanation

The correct answer is von Willebrand. Von Willebrand disease is a bleeding disorder caused by a deficiency or dysfunction of von Willebrand factor (vWF), a protein involved in blood clotting. It is characterized by prolonged bleeding time, normal platelet counts, and prolonged activated partial thromboplastin time (aPTT). Deficiencies in factors VIII, IX, and VII would result in abnormal PT and aPTT, but von Willebrand disease specifically presents with prolonged aPTT and normal PT.

Submit
24. Faciliatated diffusion:

Explanation

Facilitated diffusion is a passive transport process that allows the movement of molecules across a cell membrane along their concentration gradient. It differs from simple diffusion as it requires the assistance of specific carrier proteins embedded in the membrane. These carrier proteins bind to the molecules being transported and facilitate their movement across the membrane. Therefore, the correct answer is that facilitated diffusion requires carrier proteins.

Submit
25. The modes of infection of the following diseases are all except:

Explanation

Tetanus is not transmitted through dust droplets. Tetanus is caused by the bacterium Clostridium tetani, which is commonly found in soil, dust, and animal feces. The infection occurs when the bacteria enter the body through a wound or cut. It is not transmitted through droplets in the air.

Submit
26. True for hemochromatosis:

Explanation

Hemochromatosis is a genetic disorder characterized by excessive absorption of dietary iron, leading to iron overload in the body. Autosomal recessive inheritance means that an individual needs to inherit two copies of the mutated gene (one from each parent) in order to develop the disorder. This is supported by the fact that both males and females can be affected by hemochromatosis, indicating that it is not more common in females. Complete penetrance suggests that individuals who inherit the mutated gene will always develop the disorder, which aligns with the nature of hemochromatosis. Phlebotomy, the removal of blood, is a common treatment for hemochromatosis as it helps to reduce iron levels in the body, but it does not necessarily lead to a cure.

Submit
27. What is intermittent claudication?

Explanation

Intermittent claudication refers to pain that occurs during exercise, typically in the muscles of the legs. This pain is caused by reduced blood flow to the muscles, usually due to narrowed or blocked arteries. The pain is relieved with rest. Therefore, the correct answer is "Pain on exercise only".

Submit
28. Patient with foreign body sensation in eye and swollen knee it after leisure trip:

Explanation

Reiter's disease, also known as reactive arthritis, is a possible explanation for the patient's symptoms. It is characterized by joint inflammation, often affecting the knees, and can also cause eye inflammation and foreign body sensation. This condition is typically triggered by an infection, such as a gastrointestinal or genitourinary infection. Sarcoidosis, Behcet's disease, and SLE are all autoimmune conditions that can cause similar symptoms, but Reiter's disease is more commonly associated with joint and eye involvement after an infection.

Submit
29. Which is not involved in the intrinsic pathway?

Explanation

Factor VII is not involved in the intrinsic pathway of blood coagulation. The intrinsic pathway is initiated by the activation of factor XII, which then activates factor XI. Factor XI subsequently activates factor IX, leading to the formation of the tenase complex and ultimately the activation of factor X. Factor VII, on the other hand, is part of the extrinsic pathway, which is initiated by tissue factor and does not involve factors XII, XI, or IX.

Submit
30. A Down syndrome patient is posted for surgery. The necessary preoperative investigation to be done is:

Explanation

Echocardiography is the necessary preoperative investigation for a Down syndrome patient because individuals with Down syndrome commonly have heart defects. Echocardiography is a non-invasive test that uses sound waves to create images of the heart's structure and function, allowing doctors to identify any abnormalities or potential complications before surgery. This investigation helps in assessing the patient's cardiac health and determining the appropriate surgical approach, anesthesia, and postoperative care.

Submit
31. Marker of neural tube defect:

Explanation

Acetylcholinesterase is the correct answer because it is an enzyme that breaks down the neurotransmitter acetylcholine in the synaptic cleft, terminating the signal transmission. Neural tube defects are caused by abnormalities in the development of the neural tube during early embryonic development. The marker for neural tube defects is not specifically mentioned in the question, but acetylcholinesterase is not typically associated with neural tube defects. Therefore, it is likely that the correct answer is a mistake or the question is incomplete.

Submit
32. Incidence rate can be calculated by:

Explanation

A cohort study is a type of observational study where a group of individuals with a common characteristic or exposure are followed over a period of time to determine the incidence rate of a specific outcome or disease. In this study design, participants are initially free of the outcome of interest and are classified into different exposure groups. The incidence rate is then calculated by dividing the number of new cases of the outcome in each exposure group by the total person-time at risk in that group. This allows researchers to establish a temporal relationship between exposure and outcome and calculate the risk of developing the outcome.

Submit
33. Most recent classification according to WHO for disability:

Explanation

The most recent classification for disability according to the WHO is the International Classification of Functioning, Disability and Health (ICF). This classification system provides a framework for understanding and measuring health and disability. It focuses on the interaction between a person's health condition and their environment, and aims to provide a comprehensive understanding of functioning and disability. The ICF is widely used by healthcare professionals, researchers, and policymakers to inform decision-making and improve the lives of people with disabilities.

Submit
34. A man with blunt injury abdomen after road side accident has BP of 100/80 mm Hg, pulse 120 bpm. Respiration stabilized through air-way. Next best step in the management is:

Explanation

In a patient with blunt injury abdomen after a roadside accident, the next best step in management would be to obtain blood for cross-matching and administer intravenous fluids. The patient's low blood pressure and high heart rate indicate hypovolemia, which can be caused by internal bleeding. Blood for cross-matching is necessary to identify the patient's blood type and ensure compatibility for potential blood transfusion. Intravenous fluids are also important to restore intravascular volume and stabilize the patient before further interventions or surgery. Ventilation or rushing to the operating theater may be necessary later depending on the patient's condition, but addressing hypovolemia is the initial priority.

Submit
35. Central chemoreceptors are most sensitive to:

Explanation

Central chemoreceptors are specialized cells located in the brain that are responsible for monitoring the levels of certain gases in the blood, including carbon dioxide (CO2). These chemoreceptors are most sensitive to an increase in CO2 tension. When CO2 levels in the blood rise, it leads to an increase in carbonic acid, which then increases the concentration of hydrogen ions (H+) in the blood. This change in pH is detected by the central chemoreceptors, triggering a response to increase ventilation and remove excess CO2 from the body. Therefore, an increase in CO2 tension is the most significant stimulus for the central chemoreceptors.

Submit
36. Diastolic pressure in aorta is maintained by:

Explanation

The correct answer is the elastic recoil of the aorta. The aorta is a large, elastic artery that receives blood from the heart during systole and stretches to accommodate the volume of blood. During diastole, when the heart is at rest, the elastic fibers in the aorta recoil, helping to maintain the pressure in the aorta and ensuring continuous blood flow to the rest of the body. This elastic recoil prevents a sudden drop in pressure and helps to maintain a steady flow of blood throughout the cardiac cycle.

Submit
37. Sertoli cells are associated with:

Explanation

Sertoli cells are associated with spermiogenesis, which is the process of sperm maturation. These cells provide physical and nutritional support to developing sperm cells, as well as aid in their differentiation and maturation. They also play a role in the removal of excess cytoplasm from developing sperm cells, ensuring that they become streamlined and fully functional. Therefore, spermiogenesis is the correct association with Sertoli cells.

Submit
38. All drugs can be given to a mother with lupus who isn on 35th week of gestation except:

Explanation

Methotrexate is contraindicated in pregnant women, especially during the third trimester. It is a medication used to treat certain types of cancer and autoimmune diseases, but it can cause harm to the developing fetus. Methotrexate is known to increase the risk of miscarriage, birth defects, and developmental abnormalities. Therefore, it should not be given to a mother with lupus who is on the 35th week of gestation.

Submit
39. True for necrotizing fasciitis are all except:

Explanation

Necrotizing fasciitis is an infection that affects the fascia and subcutaneous tissue. It is commonly caused by group A beta-hemolytic streptococcus pyogenes. It typically occurs in the trunk, lower limbs, and perineum. However, the correct answer states that surgical debridement is usually not needed. This is incorrect because surgical debridement is a crucial treatment for necrotizing fasciitis. It involves removing dead tissue to prevent the spread of infection and promote healing. Therefore, the correct answer should be "Surgical debridement is usually necessary."

Submit
40. Casper's dictum is for:

Explanation

Casper's dictum refers to a principle in forensic medicine that states that the time since death can be estimated by the stage of decomposition of a body. This means that the condition of the body can provide valuable information about how long ago the person died. Therefore, Casper's dictum is specifically related to determining the time since death and not the other factors mentioned in the question such as identification of the body, mode of injury, or depth of insertion of a knife.

Submit
41. Cut off of Indian reference man (kg):

Explanation

The cutoff of the Indian reference man is 60 kg. This means that any individual who weighs less than 60 kg would not meet the criteria to be considered as the Indian reference man.

Submit
42. STEPS refers to:

Explanation

STEPS refers to assessing incidence of risk factors of non-communicable diseases. This means that STEPS is a method or tool used to evaluate and measure the occurrence or prevalence of various risk factors that contribute to the development of non-communicable diseases. It helps in identifying the factors that increase the likelihood of individuals developing such diseases, allowing for targeted interventions and preventive measures to be implemented. The other options mentioned in the question, such as assessing the incidence of non-communicable diseases, assessing Rx compliance, and assessing the rate of mortality, are not the primary focus of the STEPS approach.

Submit
43. Most important prognostic factor of colorectal carcinoma:

Explanation

The stage of the lesion is the most important prognostic factor of colorectal carcinoma. This is because the stage determines the extent of the cancer and how far it has spread. It provides crucial information about the size of the tumor, the involvement of nearby lymph nodes, and the presence of distant metastasis. The stage of the lesion helps in determining the appropriate treatment approach and predicting the patient's overall prognosis.

Submit
44. Neurofibromatosis is associated with all except:

Explanation

Neurofibromatosis is a genetic disorder that causes tumors to form on nerve tissue. It can be inherited in an autosomal dominant pattern, meaning that a person only needs to inherit one copy of the faulty gene to develop the disorder. Autosomal recessive inheritance, on the other hand, requires both copies of the gene to be faulty. Therefore, neurofibromatosis is not associated with autosomal recessive inheritance. The other options, cutaneous fibromas, cataracts, and scoliosis, are all commonly associated with neurofibromatosis.

Submit
45. True statement about RDA:

Explanation

This statement suggests that the Recommended Dietary Allowances (RDA) provide the average daily requirements of all nutrients except calories, with a margin of ± 2 standard deviations (SD). This means that the RDA takes into account the variability in nutrient needs among individuals. The RDA is used to assess the adequacy of dietary intake and applies to both healthy individuals and those who are sick.

Submit
46. What is the most common cause of death after total hip replacement in elderly lady?

Explanation

After total hip replacement surgery, elderly patients are at an increased risk of developing deep vein thrombosis (DVT). DVT occurs when a blood clot forms in the deep veins of the leg, which can then break loose and travel to the lungs, causing a pulmonary embolism. This complication is a common cause of death after hip replacement surgery, especially in elderly patients who may have other risk factors such as immobility or a history of blood clotting disorders. Infection and pneumonia are also potential complications, but DVT and pulmonary embolism are more likely and can be life-threatening if not detected and treated promptly.

Submit
47. Pt. presents with punched out lesion on X-ray withSerum Na-144, K+, Ca-12, Globulin-8.4, alls 5.4, whatwill be next investigation to do?

Explanation

Serum protein electrophoretic studies would be the next investigation to do in this case. The patient presents with a punched-out lesion on X-ray and abnormal levels of serum electrolytes, calcium, and globulin. Serum protein electrophoresis is a test that separates proteins in the blood based on their electrical charge and size, allowing for the identification of abnormal protein patterns. This test can help in diagnosing various conditions, including multiple myeloma, which can present with bone lesions and abnormal protein levels. Therefore, performing serum protein electrophoretic studies would be an appropriate next step in investigating the patient's condition.

Submit
48. Spongy part of the male urethra drains via which lymph nodes:

Explanation

The spongy part of the male urethra drains via the deep inguinal lymph nodes. The deep inguinal lymph nodes are located in the inguinal region, near the groin area. They receive lymphatic drainage from the spongy part of the male urethra, as well as from the scrotum and the penis. These lymph nodes play a role in filtering and draining lymphatic fluid from these areas, helping to remove waste products and potentially harmful substances.

Submit
49. Nutrition in the community is assessed by all/except:

Explanation

Nutrition in the community is assessed by several indicators such as 1-4 year mortality rate, height and weight of children, and birth weight below 2500 gm. However, the level of hemoglobin (Hb) in the third trimester of pregnancy is not a direct indicator of community nutrition. Hb levels in pregnancy are more specific to assessing the nutritional status of the individual pregnant woman rather than the overall community. Therefore, Hb

Submit
50. A young man with gout has synovial fluid removed. It would shows:

Explanation

The presence of MSU crystals in the synovial fluid of a young man with gout indicates a diagnosis of gout. Gout is a form of arthritis caused by the buildup of uric acid crystals in the joints. These crystals can cause severe pain, inflammation, and swelling. Removing synovial fluid and identifying the presence of MSU crystals confirms the diagnosis of gout.

Submit
51. The following drug can be given safely in pregnancy:

Explanation

Propylthiouracil is the only drug listed that is considered safe to use during pregnancy. This medication is commonly prescribed to treat hyperthyroidism in pregnant women. Both Methotrexate (MTX) and Warfarin are known to be teratogenic and can cause harm to the developing fetus. Tetracycline is also contraindicated in pregnancy as it can affect the development of the baby's teeth and bones. Therefore, Propylthiouracil is the only suitable option for pregnant women in this list.

Submit
52. Urogenital diaphragm is contributed by all except:

Explanation

The urogenital diaphragm is a muscular structure located in the pelvic floor. It plays a role in supporting the pelvic organs and maintaining continence. The sphincter urethra, perineal body, and perineal membrane are all components of the urogenital diaphragm. However, Colles' fascia is not a part of the urogenital diaphragm. Colles' fascia is a layer of connective tissue located in the perineum, but it is not directly involved in the formation of the urogenital diaphragm.

Submit
53. Organelle which plays a pivotal role in apoptosis:

Explanation

Mitochondria play a pivotal role in apoptosis because they are responsible for regulating cell death. During apoptosis, mitochondria release certain proteins that trigger a series of events leading to cell death. These proteins, such as cytochrome c, activate enzymes called caspases, which break down cellular components and ultimately lead to cell death. Therefore, the correct answer is mitochondria.

Submit
54. A girl-aged 8 years has been admitted for dialysis. She ahs serum K of 7.5 mEq/I, which is the fastest way to reduce the hyperkalemia?

Explanation

Infusion of insulin + glucose is the fastest way to reduce hyperkalemia in this case. Insulin helps to move potassium from the extracellular space into the cells, while glucose prevents hypoglycemia caused by insulin administration. This combination effectively lowers serum potassium levels. Kayexalate enema is used to treat chronic hyperkalemia, but it is not the fastest option. IV calcium gluconate is used to stabilize the myocardium in severe hyperkalemia but does not lower potassium levels. IV NaCHO3 is not a recommended treatment for hyperkalemia.

Submit
55. Bleomycin toxicity affects which type of cells:

Explanation

Bleomycin toxicity primarily affects Type-II pneumocytes. These cells are found in the alveoli of the lungs and are responsible for producing surfactant, a substance that helps to reduce surface tension and maintain the integrity of the alveoli. Bleomycin is an antineoplastic medication that can cause lung damage by inducing oxidative stress and inflammation. It specifically targets Type-II pneumocytes, leading to their injury and dysfunction. This can result in pulmonary fibrosis, a condition characterized by the excessive accumulation of scar tissue in the lungs.

Submit
56. Difference between follicular adenoma and carcinoma:

Explanation

The correct answer is "Vascular invasion." This means that the main difference between follicular adenoma and carcinoma is the presence of vascular invasion. In follicular adenoma, there is no invasion of blood vessels by the abnormal cells, whereas in carcinoma, there is invasion of blood vessels by the cancerous cells. This distinction is important in determining the aggressiveness and potential for metastasis of the tumor.

Submit
57. Positive hepatojugular reflux is found in conditions except:

Explanation

Positive hepatojugular reflux is a physical finding that indicates increased venous pressure in the liver. It is commonly seen in conditions such as tricuspid regurgitation and right heart failure, where there is impaired blood flow from the right side of the heart. However, decreased afterload refers to a decrease in the resistance that the heart must overcome to eject blood. This would not cause increased venous pressure in the liver and therefore would not be associated with positive hepatojugular reflux.

Submit
58. A neonate with focal skin lesions and hypoplastic limbs causative agent:

Explanation

Herpes Zoster is the correct answer because it is a viral infection caused by the varicella-zoster virus. It typically presents with painful skin lesions in a dermatomal distribution, which can occur in neonates. Hypoplastic limbs, however, are not commonly associated with Herpes Zoster.

Submit
59. A 29 years old person known diabetic on OHA since 3 years. He has lost weight. Never had DKA. His grand father is diabetic. His father is nondiabetic. Which is the likely diagnosis:

Explanation

Based on the information provided, the most likely diagnosis for the 29-year-old person is DM Type II. This is because the person is known to be diabetic, has been on oral hypoglycemic agents (OHA) for 3 years, and has a family history of diabetes (grandfather). Additionally, the fact that the person has lost weight and never had diabetic ketoacidosis (DKA) suggests that they are not experiencing the more severe symptoms typically associated with DM Type I or pancreatic diabetes. Therefore, the most likely explanation is DM Type II.

Submit
60. Angiotensin II causes all except:

Explanation

Angiotensin II is a hormone that is primarily known for its vasoconstrictive effects, meaning it causes blood vessels to narrow. This narrowing of blood vessels leads to an increase in blood pressure. Therefore, it is logical to conclude that angiotensin II would not cause vasodilation, as this would contradict its vasoconstrictive properties.

Submit
61. Disease, which permanently alters finger print: (DNB JUNE)

Explanation

Leprosy is a disease caused by the bacteria Mycobacterium leprae that primarily affects the skin and peripheral nerves. It can lead to permanent changes in the skin, including thickening, discoloration, and loss of sensation. In some cases, leprosy can cause damage to the sweat glands and oil glands, which can result in changes to the fingerprints. Therefore, leprosy is the correct answer as it is the only disease listed that can permanently alter fingerprints.

Submit
62. Patient with pancreatic transplant with urinary drainage. Monitoring will be done by:

Explanation

After a pancreatic transplant, the patient may have a urinary drainage system in place. Monitoring urine amylase levels is important in this case because it can indicate any complications or rejection of the transplant. Elevated levels of urine amylase may suggest pancreatic injury or dysfunction. Blood amylase, serum glucose levels, and serum lipase levels may also be monitored, but urine amylase is specifically mentioned in the question as the correct answer for monitoring in a patient with pancreatic transplant and urinary drainage.

Submit
63. Acetyl CoA can be directly converted to all except:

Explanation

Acetyl CoA can be directly converted to fatty acids, ketone bodies, and cholesterol through various metabolic pathways. However, it cannot be directly converted to glucose. Glucose is primarily synthesized through a process called gluconeogenesis, which involves the conversion of non-carbohydrate sources like amino acids and lactate into glucose. Acetyl CoA is an important molecule in energy metabolism, but it cannot be converted back into glucose.

Submit
64. In surgical staging of ovarian Ca, all are done except:

Explanation

In surgical staging of ovarian cancer, various procedures are performed to determine the extent of the disease. These include peritoneal washing, omental biopsy, and palpation of organs. However, a liver biopsy is not typically part of the staging process for ovarian cancer. This is because the primary focus of staging is to assess the spread of the cancer within the pelvis and abdomen, and the liver is not a common site for ovarian cancer metastasis. Therefore, a liver biopsy is not necessary for surgical staging of ovarian cancer.

Submit
65. Best marker for stratification of Carciovascular disease risk:

Explanation

Lipoprotein (a) is the best marker for stratification of cardiovascular disease risk because it is a type of lipoprotein that is similar to LDL cholesterol and has been found to be strongly associated with an increased risk of developing cardiovascular disease. Elevated levels of lipoprotein (a) in the blood can lead to the formation of blood clots and the development of atherosclerosis, which are both major contributors to cardiovascular disease. Therefore, measuring lipoprotein (a) levels can provide valuable information about an individual's risk of developing cardiovascular disease.

Submit
66. C wave in JVP indicates:

Explanation

The bulging of the tricuspid valve is indicated by a C wave in the jugular venous pulse (JVP). This occurs during ventricular systole when the right ventricle contracts and forces blood against the closed tricuspid valve, causing it to bulge into the right atrium. This bulging can be observed as a prominent upward deflection in the JVP waveform.

Submit
67. Mifepristone is used in the management of:

Explanation

Mifepristone is used in the management of ectopic pregnancy. Ectopic pregnancy occurs when a fertilized egg implants outside the uterus, usually in the fallopian tube. Mifepristone is a medication that blocks the action of progesterone, a hormone necessary for pregnancy. By blocking progesterone, mifepristone can help to terminate an ectopic pregnancy. It is important to diagnose and treat ectopic pregnancies promptly, as they can be life-threatening if left untreated.

Submit
68. The following are the complications of the HIV infection except:

Explanation

HIV infection can lead to various complications, including cardiac tamponade, pericardial effusion, and cardiomyopathy. However, aortic aneurysm is not typically associated with HIV infection. Aortic aneurysms are more commonly caused by atherosclerosis, hypertension, and genetic factors. Therefore, the correct answer is aortic aneurysm.

Submit
69. About renal physiology all are true except:

Explanation

The statement "5% cardiac output is received by kidney" is incorrect. The kidneys actually receive approximately 20-25% of the cardiac output. This is because the kidneys play a crucial role in filtering blood and maintaining fluid balance in the body. The high blood flow to the kidneys allows for efficient filtration and excretion of waste products.

Submit
70. Oxygen demand of heart:

Explanation

The oxygen demand of the heart increases proportionately with heart rate because as the heart beats faster, it needs to pump more blood to meet the body's demand for oxygen. This increased pumping requires more energy and therefore more oxygen.

Submit
71. In simple random sampling:

Explanation

In simple random sampling, all individuals in the population have an equal chance of being selected. This means that every member of the population has the same probability of being chosen as a sample. This method ensures that the sample is representative of the entire population and reduces bias. It is suitable for large and heterogeneous populations where there is no need to consider individual characteristics or when a complete sampling frame is not available.

Submit
72. True regarding DMPA include the following except:

Explanation

DMPA, or depot medroxyprogesterone acetate, is a form of birth control that is administered through injection. It has a failure rate of 0.3%, meaning that it is highly effective in preventing pregnancy. It does not have a protective effect on the cervical and endometrial cancer. However, it can be given to patients with seizures as it does not increase the risk of seizures. Additionally, DMPA can be used in the treatment of menorrhagia, a condition characterized by heavy or prolonged menstrual bleeding.

Submit
73. Characteristic of HbS is all except:

Explanation

The characteristic of HbS that is not true is the increase in sticky patch due to substitution of a nonpolar residue by a polar residue. HbS is a variant of hemoglobin where there is a single base substitution, changing glutamine to valine. Heterozygotes with HbS are actually protected against malaria. The substitution of a nonpolar residue by a polar residue does not lead to an increase in sticky patch, but rather affects the overall structure and function of the protein.

Submit
74. Finnish type of nephritic syndrome is associated with:

Explanation

Finnish type of nephritic syndrome is associated with nephrin. Nephrin is a protein that plays a crucial role in maintaining the structure and function of the glomerular filtration barrier in the kidneys. Mutations in the nephrin gene can lead to a disruption of this barrier, resulting in increased permeability of the glomerulus and the development of nephritic syndrome. This syndrome is characterized by inflammation of the glomeruli, leading to symptoms such as proteinuria, hematuria, and decreased renal function.

Submit
75. Ca breast is not discovered in mammography in young woman due to:

Explanation

Young women often have denser breast tissue, which can make it difficult to detect abnormalities on a mammogram. Dense glandular tissue appears white on a mammogram, similar to tumors or other abnormalities, making it harder to distinguish between them. This can lead to false-negative results, where breast cancer may go undetected. Regular breast exams and other imaging techniques may be recommended for young women with dense breast tissue to ensure early detection of any abnormalities.

Submit
76. Ifosfamide is:

Explanation

Ifosfamide is classified as an alkylating agent. Alkylating agents are a class of chemotherapy drugs that work by attaching alkyl groups to DNA molecules, which prevents the DNA from replicating and ultimately leads to cell death. Ifosfamide is commonly used in the treatment of various types of cancer, including testicular, ovarian, and lung cancer. It is often used in combination with other chemotherapy drugs to enhance its effectiveness.

Submit
77. Which structure is just lateral to anterior perforated substance?

Explanation

The structure that is just lateral to the anterior perforated substance is the limen insulae.

Submit
78. Causes of the sigmoid shape of the oxygen-hemoglobin dissociation curve (O2) is:

Explanation

The sigmoid shape of the oxygen-hemoglobin dissociation curve is due to the cooperative binding of oxygen molecules to hemoglobin. When one oxygen molecule binds to a subunit of hemoglobin, it induces conformational changes that increase the affinity of the remaining subunits for oxygen. This allows hemoglobin to bind more oxygen molecules more easily, resulting in a steep increase in oxygen saturation at higher oxygen partial pressures. This cooperative binding behavior is responsible for the sigmoid shape of the curve. The Bohr effect and Haldane effect are related to the regulation of oxygen binding and release by factors such as pH and carbon dioxide, but they do not directly explain the sigmoid shape of the curve.

Submit
79. Test for assessment of the mucosal function of GIT:

Explanation

The D-Xylose test is a diagnostic test used to assess the mucosal function of the gastrointestinal tract (GIT). It involves administering a dose of D-Xylose, a sugar that is normally absorbed by the small intestine. The test measures the levels of D-Xylose in the urine or blood to determine if the small intestine is properly absorbing nutrients. If the levels are low, it may indicate malabsorption or mucosal damage in the GIT. Therefore, the D-Xylose test is an appropriate choice for assessing mucosal function in the GIT.

Submit
80. If a patient with severe hyperglycemia is given IV insulin, which of the following can occur?

Explanation

When a patient with severe hyperglycemia is given IV insulin, it can cause a shift of potassium from the extracellular fluid into the intracellular fluid. This shift of potassium can lead to decreased serum potassium levels, resulting in hypokalemia. Insulin stimulates the uptake of glucose and potassium into cells, therefore, as glucose enters the cells, so does potassium. This can be further exacerbated by insulin's ability to stimulate the activity of the sodium-potassium pump, leading to increased potassium uptake by cells and subsequent hypokalemia.

Submit
81. A young man met with an accident leading to loss of hearing in the right ear. On otoscopy examination the tympanic membrane was intact. Pure tone audiometry showed AB gap of 55 dB on the right ear with normal cochlear reserve. Which of the following will be probable tympanometry finding?

Explanation

The probable tympanometry finding in this case would be an Ad type tympanogram. This is because the tympanic membrane is intact, but there is a loss of hearing in the right ear. An Ad type tympanogram indicates a normal middle ear pressure with reduced compliance, which could be caused by a stiffened or immobilized tympanic membrane. This finding is consistent with the intact tympanic membrane and the loss of hearing in the right ear.

Submit
82. All of the following arteries supply medulla except:

Explanation

The medulla is supplied by several arteries, including the posterior inferior cerebellar artery, basilar artery, and anterior spinal artery. However, the bulbar artery does not supply the medulla.

Submit
83. Which structure is just lateral to anterior perforated substance?

Explanation

The structure that is just lateral to the anterior perforated substance is the Limen insulae.

Submit
84. Superior gluteal nerve supplies all except:

Explanation

The superior gluteal nerve supplies the gluteus minimus, gluteus medius, and tensor fascia lata muscles. However, it does not supply the gluteus maximus muscle.

Submit
85. Appetite suppressors are all except:

Explanation

Neuropeptide Y is not an appetite suppressor. It is actually a neurotransmitter that stimulates appetite and increases food intake. Therefore, it is not included in the list of appetite suppressors.

Submit
86. In obstructive azoospermia:

Explanation

In obstructive azoospermia, the obstruction in the reproductive tract prevents the release of sperm, but it does not affect the production of hormones by the pituitary gland. Therefore, the levels of follicle-stimulating hormone (FSH) and luteinizing hormone (LH) remain normal.

Submit
87. Minimum duration between onset of symptoms and death is seen in:

Explanation

Postpartum hemorrhage (PPH) is the excessive bleeding that occurs after childbirth. It is a life-threatening condition that can lead to death if not promptly treated. The minimum duration between the onset of symptoms (excessive bleeding) and death is likely to be shorter in PPH compared to the other options listed. This is because PPH involves rapid and significant blood loss, which can quickly lead to severe complications and death if not managed urgently.

Submit
88. Maltese cross is associated with:

Explanation

The Maltese cross is a symbol associated with Babesia microti. Babesia microti is a parasitic protozoan that causes babesiosis, a tick-borne illness. The Maltese cross is often used to represent this parasite because its shape resembles the cross used by the Knights of Malta, who were known for their efforts in treating and caring for those affected by the disease.

Submit
89. Heat hematoma differs from blunt trauma by:

Explanation

The correct answer is "Margins are irregular." In heat hematoma, the margins or edges of the hematoma are irregular, meaning they are not smooth or uniform. This is in contrast to blunt trauma, where the margins of the hematoma are typically smooth. The irregular margins in heat hematoma may be indicative of underlying tissue damage or inflammation caused by the heat.

Submit
90. Splicing is associated with:

Explanation

Splicing is a process in molecular biology where introns (non-coding regions) are removed from pre-mRNA and exons (coding regions) are joined together to form mature mRNA. This process is specifically associated with Sn RNA (small nuclear RNA). Sn RNA molecules are involved in the formation of spliceosomes, which are complexes responsible for the splicing of pre-mRNA. Therefore, the correct answer is Sn RNA.

Submit
91. In a standard distribution curve:

Explanation

In a standard distribution curve, the mean represents the average value of the data set, while the median represents the middle value. The fact that the mean is equal to the median suggests that the data is symmetrically distributed around the center. This means that the values on both sides of the distribution are balanced, resulting in the mean and median being the same.

Submit
92. What is specific for GIST?

Explanation

CD 117 is specific for GIST. This means that CD 117 is a marker or protein that is commonly found in gastrointestinal stromal tumors (GIST). It is used as a diagnostic tool to identify and differentiate GIST from other types of tumors. CD 117 is a receptor tyrosine kinase protein that is expressed on the surface of GIST cells. Its presence helps in confirming the diagnosis of GIST and can also be used as a target for certain treatments, such as tyrosine kinase inhibitors.

Submit
93. When one gene is inherited from one parent only, it is known as:

Explanation

Genomic imprinting refers to the phenomenon where certain genes are expressed in a parent-of-origin-specific manner. This means that the expression of these genes is determined by whether they are inherited from the mother or the father. In other words, when one gene is inherited from one parent only, it is known as genomic imprinting. This process plays a crucial role in regulating gene expression and can have important implications for development and disease.

Submit
94. What is affected in HbS (Hemoglobin S)?

Explanation

HbS (Hemoglobin S) refers to a genetic variant of hemoglobin, the protein responsible for carrying oxygen in red blood cells. This variant is caused by a mutation in the gene that codes for hemoglobin, resulting in a change in the structure of the protein. Specifically, in HbS, the mutation causes the amino acid glutamic acid to be replaced by valine. This change in structure affects the solubility of hemoglobin, causing it to form insoluble fibers when deoxygenated. These fibers can distort the shape of red blood cells, leading to their sickling and causing various health problems associated with sickle cell disease.

Submit
95. All is true for Sternberg canal except:

Explanation

The Sternberg canal is a small canal located in the sphenoid bone of the skull. It is anterior and medial to the foramen rotundum, which is a passage for a nerve called the maxillary nerve. The canal is not posterior and lateral to the foramen rotundum. The other statements are true about the Sternberg canal: it can be a cause of intra-sphenoidal meningocele, it can carry infection to the sphenoidal sinus, and it is located anterior and medial to the foramen rotundum.

Submit
96. Coagulative necrosis is found in which infection?

Explanation

Coagulative necrosis is a type of cell death characterized by the preservation of tissue architecture due to the denaturation of structural proteins. It is commonly found in tuberculosis (TB) infection. TB is caused by the bacteria Mycobacterium tuberculosis, which primarily affects the lungs. The infection leads to the formation of granulomas, which are composed of necrotic tissue surrounded by a wall of immune cells. Within these granulomas, coagulative necrosis occurs, resulting in the characteristic caseous necrotic material. Therefore, TB is the correct answer for the infection associated with coagulative necrosis.

Submit
97. Immediately after eating a man developed lump in throat, pruritis, dyspnea, cyanosis and not being able to breath after few hours. Most probable cause is:

Explanation

The symptoms described in the question, such as a lump in the throat, pruritis (itching), dyspnea (difficulty breathing), cyanosis (bluish discoloration of the skin), and not being able to breathe, are consistent with an allergic or hypersensitivity reaction. Angioneurotic edema is a type of allergic reaction that can cause swelling in various parts of the body, including the throat, leading to difficulty breathing. Therefore, the most probable cause of the symptoms described in the question is a hypersensitivity reaction. MI (myocardial infarction) and food stuck in the larynx are not likely causes based on the symptoms provided.

Submit
98. All are the side effects of tacrolimus except:

Explanation

Tacrolimus is an immunosuppressive medication used to prevent organ rejection after transplantation. It works by suppressing the immune system, but it can also cause various side effects. Hepatoxicity refers to liver damage, nephrotoxicity refers to kidney damage, and neurotoxicity refers to damage to the nervous system. These are known side effects of tacrolimus. However, ototoxicity, which refers to damage to the ears and hearing loss, is not commonly associated with tacrolimus use. Therefore, ototoxicity is the correct answer as it is not a known side effect of tacrolimus.

Submit
99. A most common sign of acute hypoxia in neonates:

Explanation

Bradycardia is a common sign of acute hypoxia in neonates. When the body is deprived of oxygen, the heart rate slows down as a compensatory mechanism. This is because the body tries to conserve oxygen by reducing the heart's workload. Therefore, bradycardia is a reliable indicator of hypoxia in neonates.

Submit
100. Which of the following is not true about JRA?

Explanation

Raynaud's phenomenon is not true about JRA. JRA, or Juvenile Rheumatoid Arthritis, is a chronic inflammatory disorder that primarily affects children. Common symptoms include fever, rheumatoid nodules, and uveitis, which is inflammation of the middle layer of the eye. However, Raynaud's phenomenon, which is characterized by episodes of color changes in the fingers and toes due to cold or stress, is not typically associated with JRA.

Submit
101. Following is the NCCT head of a 40-year-old patient who is unconscious after a blunt head injury. The finding is suggestive of

Explanation

The NCCT head of a 40-year-old patient who is unconscious after a blunt head injury is suggestive of an extradural hematoma. This is because an extradural hematoma is characterized by a collection of blood between the skull and the outermost layer of the brain, known as the dura mater. The CT scan would typically show a biconvex or lens-shaped hematoma that is located between the skull and the brain, causing compression of the brain tissue. This finding is consistent with an extradural hematoma rather than other types of bleeding such as subdural hematoma, subarachnoid bleed, or intracerebral bleed.

Submit
102. What is not true about nontyphoidal Salmonella?

Explanation

Blood culture is not more sensitive than stool culture for detecting nontyphoidal Salmonella. Stool culture is the preferred method for diagnosing nontyphoidal Salmonella infections as the bacteria is typically shed in the feces. Blood culture may be used in severe cases or in immunocompromised individuals, but it is not more sensitive than stool culture.

Submit
103. After 48 hours of fasting insulin receptors are down regulated in:

Explanation

After 48 hours of fasting, insulin receptors are downregulated in adipocytes. This means that the number of insulin receptors on the surface of adipocytes decreases. Insulin receptors are responsible for binding to insulin and allowing it to exert its effects on cells. Downregulation of insulin receptors in adipocytes suggests that these cells become less responsive to insulin signaling. This may be a mechanism to conserve energy during fasting, as insulin promotes the uptake and storage of glucose and fatty acids in adipocytes.

Submit
104. CAP in Lac Operon is an example of:

Explanation

CAP (catabolite activator protein) in the Lac Operon is an example of a positive regulator. CAP binds to a specific DNA sequence upstream of the lac operon, known as the CAP site, and enhances the transcription of the operon. In the absence of glucose, CAP is activated by binding to cyclic AMP (cAMP), which allows it to bind to the CAP site and facilitate RNA polymerase binding and transcription of the lac operon. Therefore, CAP acts as a positive regulator by increasing gene expression in the presence of cAMP.

Submit
105. Use of tamoxifen in HRT does not lead to

Explanation

Tamoxifen is a selective estrogen receptor modulator (SERM) commonly used in hormone replacement therapy (HRT) for breast cancer patients. It has been found to significantly reduce the risk of developing contralateral breast cancer, which refers to cancer occurring in the opposite breast. This is because tamoxifen acts by blocking the estrogen receptors in breast tissue, inhibiting the growth of cancer cells. Therefore, the use of tamoxifen in HRT does not lead to an increased risk of developing cancer in the contralateral breast.

Submit
106. A patients is given tacrolimus, which antibiotic should not be given to him?

Explanation

Rifampicin should not be given to a patient who is taking tacrolimus. Rifampicin is known to induce the activity of liver enzymes, which can lead to a decrease in the blood levels of tacrolimus. This can result in reduced effectiveness of tacrolimus in preventing organ rejection in transplant patients. Therefore, it is important to avoid the concomitant use of rifampicin and tacrolimus.

Submit
107. Cell-cell interaction occurs by all of the following mechanisms except:

Explanation

Cell-cell interaction can occur through various mechanisms such as hormones, gap junctions, and neurotransmitters released at the synapse. G-protein receptors, however, are not involved in direct cell-cell communication. G-protein receptors are membrane proteins that are activated by ligands, such as hormones or neurotransmitters, and initiate intracellular signaling pathways. While they play a crucial role in transmitting signals within a cell, they do not directly mediate interactions between cells.

Submit
108. The status of fluid in distal convoluted tubule is all except:

Explanation

The status of fluid in the distal convoluted tubule is not always hypotonic. The distal convoluted tubule is responsible for the reabsorption of water and electrolytes from the filtrate. The amount of water reabsorbed varies depending on the body's needs and hormonal signals such as antidiuretic hormone (ADH). Therefore, the fluid in the distal convoluted tubule can be hypotonic, hypertonic, or isotonic depending on the circumstances.

Submit
109. True about deglutition persistalsis of esophagus:

Explanation

Primary peristalsis is the initial wave of muscular contractions that occurs in the esophagus after swallowing. It is an involuntary process that helps propel the food bolus from the mouth to the stomach. This wave is initiated by the swallowing reflex and is essential for the normal movement of food through the esophagus. Secondary peristalsis occurs when there is a blockage or incomplete clearance of the esophagus, and it serves as a backup mechanism to clear the obstruction. Tertiary and quarternary peristalsis are not recognized terms in relation to the esophagus.

Submit
110. Potential for Reperfusion of the tissue, post-infarct can be assessed by:

Explanation

Thallium scan is used to assess the potential for reperfusion of the tissue post-infarct. Thallium is a radioactive substance that is injected into the bloodstream and taken up by the heart muscle. Areas of the heart that have reduced blood flow due to a blockage or infarction will show decreased thallium uptake. This can indicate areas of irreversible damage where reperfusion may not be possible. On the other hand, areas with normal or increased thallium uptake suggest viable tissue that may benefit from reperfusion therapy. Therefore, a Thallium scan is a useful tool for evaluating the potential for tissue reperfusion after an infarct.

Submit
111. Best material for below inguinal arterial graft:

Explanation

The best material for an inguinal arterial graft is the saphenous vein used in an upside-down position. This is because the saphenous vein has good long-term patency rates and is readily available in the patient's own body. By using it in an upside-down position, the valves within the vein are eliminated, reducing the risk of graft failure. PTFE, Dacron carmustine, and Teflon are alternative graft materials, but they do not offer the same advantages as the saphenous vein.

Submit
112. The patient has chronic arterial obstruction. He presents with:

Explanation

The patient is experiencing intermittent claudication, which is a symptom of chronic arterial obstruction. Intermittent claudication refers to pain or cramping in the muscles of the legs that occurs during physical activity and is relieved with rest. This is a common symptom of peripheral artery disease (PAD), which is caused by a narrowing or blockage of the arteries that supply blood to the legs. Gangrene and rest pains are also associated with chronic arterial obstruction, but the patient specifically presents with intermittent claudication.

Submit
113. Coombs positive hemolytic anemia associated with:

Explanation

Coombs positive hemolytic anemia is a condition characterized by the destruction of red blood cells due to the presence of antibodies that target these cells. Systemic lupus erythematosus (SLE) is an autoimmune disease that can cause the production of these antibodies, leading to the development of hemolytic anemia. Therefore, SLE is associated with Coombs positive hemolytic anemia.

Submit
114. About Hepatitis C, which is true?

Explanation

The correct answer is "MC indication for liver transplant." This means that Hepatitis C is the most common indication or reason for a liver transplant. This suggests that Hepatitis C can severely damage the liver to the point where a transplant is necessary for survival.

Submit
115. Which of these is an FDA approved indication for use of modafinil as an adjunct?

Explanation

Modafinil is approved by the FDA for the treatment of narcolepsy. Narcolepsy is a sleep disorder characterized by excessive daytime sleepiness and uncontrollable episodes of falling asleep. Modafinil helps to promote wakefulness and reduce excessive sleepiness in individuals with narcolepsy. It is not approved for the treatment of major depression, obstructive sleep apnea, or shift work disorder.

Submit
116. Triple assessment for Ca breast is:

Explanation

The triple assessment for Ca breast involves three components: clinical examination, mammogram, and FNAC/biopsy. Clinical examination allows for a physical examination of the breast to detect any abnormalities. Mammogram is a radiographic examination that uses X-rays to identify any suspicious changes in breast tissue. FNAC (fine-needle aspiration cytology) or biopsy involves the collection of cells or tissue from the breast for laboratory analysis to determine if cancer is present. Therefore, the correct answer is clinical examination, mammogram, and FNAC/biopsy.

Submit
117. Primary Skin lesions are seen in all except:

Explanation

Primary skin lesions are the initial or basic skin abnormalities that arise from a specific disease or disorder. Bowen's disease is a type of skin condition characterized by the presence of squamous cell carcinoma in situ. Reiter's syndrome, psoriasis, and lichen planus are all examples of conditions that can present with primary skin lesions. Therefore, Bowen's disease is the exception in this list as it is a primary skin lesion itself.

Submit
118. In a program the end point of all activities which may not be operationally measurable is called?

Explanation

An objective refers to the end point of all activities in a program, even those that may not be operationally measurable. It represents the desired outcome or result that the program aims to achieve. Objectives provide a clear direction and purpose for the program, guiding the actions and decisions made throughout its implementation. They help to define the overall vision and establish the criteria for success.

Submit
119. Myodesis is contraindicated in:

Explanation

Myodesis is a surgical procedure that involves suturing or anchoring muscles to adjacent structures. It is contraindicated in cases of ischemia, which refers to inadequate blood supply to a particular organ or tissue. Ischemia can lead to tissue damage or death, and performing myodesis in such cases could exacerbate the ischemic condition and cause further harm. Therefore, it is important to avoid myodesis in patients with ischemia to prevent potential complications.

Submit
120. Meningitis spread from CNS to Inner ear through:

Explanation

The cochlear aqueduct is a small canal that connects the inner ear to the subarachnoid space surrounding the brain and spinal cord. Meningitis, an infection that affects the membranes surrounding the brain and spinal cord, can spread from the central nervous system (CNS) to the inner ear through this pathway. This allows the infection to reach the structures responsible for hearing, such as the cochlea, leading to potential hearing loss or other auditory symptoms.

Submit
121. Adrenal adenoma on CT which is not true:

Explanation

This statement is not true because in adrenal adenoma on CT, the contrast typically appears early and washes out early, not late. Adrenal adenomas have a characteristic imaging pattern known as "rapid washout," where the contrast enhancement is high in the arterial phase and rapidly decreases in the venous and delayed phases. This is in contrast to other adrenal lesions like adrenal carcinomas, which show persistent enhancement in the delayed phases.

Submit
122. Aldosterone leads to all except:

Explanation

Aldosterone is a hormone produced by the adrenal glands that helps regulate sodium and potassium levels in the body. It promotes the reabsorption of sodium and the excretion of potassium in the kidneys. This leads to increased sodium levels in the blood (hypernatremia) and decreased potassium levels (hypokalemia). Additionally, aldosterone causes water retention, which can contribute to hypertension (HTN). However, aldosterone does not directly cause metabolic acidosis, which is characterized by an imbalance of acid and base in the body.

Submit
123. Posterior superior alveolar nerve is a branch of:

Explanation

The posterior superior alveolar nerve is a branch of the maxillary nerve. The maxillary nerve is one of the three main branches of the trigeminal nerve, which is responsible for sensory innervation of the face. The maxillary nerve supplies sensory information to the upper teeth and gums, as well as the upper lip, cheek, and part of the nasal cavity. Therefore, it makes sense that the posterior superior alveolar nerve, which innervates the upper molars and premolars, would be a branch of the maxillary nerve.

Submit
124. GALT (Gut Associated Lymphoid Tissue) is present in:

Explanation

GALT (Gut Associated Lymphoid Tissue) is present in the lamina propria. The lamina propria is a layer of loose connective tissue found directly beneath the epithelial lining of the gastrointestinal tract. It contains numerous lymphocytes, plasma cells, and other immune cells, making it an important component of the body's immune defense system in the gut. GALT plays a crucial role in monitoring and responding to antigens present in the gut, helping to maintain immune homeostasis and protect against pathogens.

Submit
125. Paravertebral black, can extend into all except:

Explanation

The paravertebral black can extend into the sub-arachnoid space, epidural space, and intercostal space. However, it cannot extend into the sub and inferior paravertebral space.

Submit
126. E.coli 0157 is cultured not on:

Explanation

Sorbitol MacConkey agar is not used for culturing E.coli 0157 because this specific strain of E.coli does not ferment sorbitol, which is a sugar present in the agar. This agar is selective for other types of E.coli that can ferment sorbitol, allowing them to grow and produce pink colonies. However, E.coli 0157 cannot ferment sorbitol, so it does not grow on this agar.

Submit
127. Pasturella multocida infection is due to:

Explanation

Pasturella multocida infection is caused by the bacteria Pasturella multocida, which is commonly found in the mouths of animals. When an animal bites a human, the bacteria can be transferred into the wound, leading to an infection. This infection is commonly known as a "bite wound infection" and can cause symptoms such as redness, swelling, and pain at the site of the bite. Therefore, the correct answer is animal bite.

Submit
128. MC cause of superficial thrombophlebitis:

Explanation

Superficial thrombophlebitis is inflammation and blood clot formation in a superficial vein near the surface of the skin. One of the main causes of this condition is IV injections, which can cause irritation and damage to the vein, leading to inflammation and clot formation. The insertion of a needle and the administration of medication or fluids through an IV can disrupt the normal blood flow in the vein and trigger the inflammatory response. Therefore, IV injections are a common cause of superficial thrombophlebitis.

Submit
129. False about Aedes aegypti is:

Explanation

Aedes aegypti is a species of mosquito that is known to transmit diseases such as dengue. It is also known for its repeated biting behavior. However, the statement that eggs can survive without water for more than 7-8 days is false. Mosquito eggs, including those of Aedes aegypti, require water to hatch and develop. Without water, the eggs will not be able to survive and will not hatch.

Submit
130. Baby friendly hospital initiative all except:

Explanation

The Baby Friendly Hospital Initiative promotes practices that support and encourage breastfeeding. Initiating breastfeeding within 4 hours of delivery is not a recommended practice as it may interfere with the natural process of breastfeeding and bonding between the mother and baby immediately after birth. It is recommended to initiate breastfeeding as soon as possible after birth, ideally within the first hour, to ensure successful breastfeeding initiation and establish a good milk supply.

Submit
131. Popliteal pulse is not felt clearly because:

Explanation

The popliteal pulse is not felt clearly because it is deep seated. This means that the pulse is located deep within the tissues and is not easily accessible or detectable by touch. Unlike other pulses that are more superficial and can be felt more easily, the popliteal pulse is not as prominent and requires more pressure or skill to be felt.

Submit
132. Regarding angioneurotic edema, what is not true?

Explanation

Angioneurotic edema is characterized by non-pitting edema, meaning that it does not leave an indentation when pressure is applied. This edema typically affects the face, including the lips, but it is not associated with pitting. The other options mentioned in the question are all true statements about angioneurotic edema.

Submit
133. Dengue hemorrhagic fever occurs in:

Explanation

Dengue hemorrhagic fever can occur when a person who has previously been infected with one serotype of the dengue virus is infected with a different serotype. This is known as re-infection from other viruses. When a person is infected with a new serotype, their immune response may not be able to effectively fight off the virus, leading to more severe symptoms and potentially dengue hemorrhagic fever.

Submit
134. The creamy fishy odor is caused by:

Explanation

Gardnerella is the correct answer because it is a bacterium that can cause bacterial vaginosis, a common vaginal infection. One of the symptoms of bacterial vaginosis is a fishy odor, which is often described as creamy. Trichomonas is a parasite that can also cause a fishy odor, but it is not creamy. Candida is a fungus that can cause a yeast infection, but it does not typically cause a fishy odor. Chlamydia is a sexually transmitted infection that does not typically cause a fishy odor.

Submit
135. Non-invasive diarrhea caused by all except:

Explanation

Non-invasive diarrhea is a type of diarrhea that does not involve invasion or penetration of the intestinal wall by the causative agent. Shigella, EIEC, and Y. enterocolitica are all known to cause invasive diarrhea, meaning they are able to invade the intestinal wall and cause more severe symptoms. B. cereus, on the other hand, is known to cause non-invasive diarrhea. Therefore, B. cereus is the correct answer as it does not cause invasive diarrhea like the other options.

Submit
136. Muscle's blood supply increases during exercise due to:

Explanation

During exercise, the muscles require more oxygen and nutrients to function properly. As a result, there is an increased production of active metabolites such as lactic acid. These metabolites act as vasodilators, causing the blood vessels in the muscles to widen or dilate. This dilation allows for more blood to flow through the muscles, delivering the necessary oxygen and nutrients. Therefore, the accumulation of active metabolites is the reason why the muscle's blood supply increases during exercise.

Submit
137. Energy expenditure in resting state depends upon:

Explanation

In resting state, the body requires energy to perform basic functions such as breathing, circulating blood, and maintaining body temperature. Lean body mass refers to the weight of muscles, organs, and bones, which are metabolically active tissues that require energy even at rest. Therefore, the energy expenditure in resting state is dependent on lean body mass. Adipose tissue (fat) does not require as much energy at rest, and heart rate alone does not directly determine energy expenditure.

Submit
138. Regarding crude birth rate all are true except:

Explanation

The given statement is incorrect because the crude birth rate is not a better measure of fertility than the general fertility rate. The general fertility rate takes into account the age distribution of women in the population, which is important in understanding fertility patterns. The crude birth rate, on the other hand, does not consider age distribution and only provides a simple measure of the number of births in a population. Therefore, the statement that it is a better measure of fertility than the general fertility rate is not true.

Submit
139. Primary amenorrhea is not seen in:

Explanation

Primary amenorrhea refers to the absence of menstruation by the age of 16 in girls who have otherwise developed secondary sexual characteristics. Sheehan's syndrome is a condition caused by severe postpartum hemorrhage, leading to pituitary gland infarction and subsequent hormonal deficiencies. Kallmann's syndrome is a genetic disorder characterized by delayed or absent puberty and impaired sense of smell. Mayer Rokitsansky Kuster Hauser syndrome is a congenital disorder characterized by the absence or underdevelopment of the uterus and vagina. Turner syndrome is a genetic disorder in females characterized by the absence or underdevelopment of the ovaries, leading to infertility and other physical abnormalities. Sheehan's syndrome is not associated with primary amenorrhea.

Submit
140. All is true about trochlear nerve except:

Explanation

The trochlear nerve is the only cranial nerve that arises from the dorsal aspect of the brainstem, making the statement "Arise from dorsal aspect" true. It also has the longest intracranial course among all the cranial nerves. Additionally, it supplies the contralateral superior oblique muscle, not the ipsilateral one. Therefore, the statement "Supplies ipsilateral superior oblique" is false. Lastly, the trochlear nerve does arise from outside the common tendinous ring, which is a true statement.

Submit
141. Calcification of posterior spinal ligament is best diagnosed by:

Explanation

CT (computed tomography) is the best diagnostic tool for calcification of the posterior spinal ligament. CT scans provide detailed images of the bones and soft tissues, allowing for the detection and evaluation of calcifications. MRI (magnetic resonance imaging) is useful for visualizing soft tissues, but it may not be as effective in detecting calcifications. X-ray imaging can show bony changes, but it may not provide sufficient detail for diagnosing calcification in the posterior spinal ligament. USG (ultrasound) is not the preferred imaging modality for evaluating calcifications in this area.

Submit
142. Psammoma bodies are seen in all except:

Explanation

Psammoma bodies are calcified structures that can be seen in certain types of tumors, such as papillary carcinoma of the thyroid and meningioma. However, they are not typically seen in follicular carcinoma of the thyroid or serous cystadenoma.

Submit
143. All are pain sensitive area of brain except:

Explanation

The choroid plexus is not a pain-sensitive area of the brain. It is a network of blood vessels located within the ventricles of the brain, responsible for producing cerebrospinal fluid. While the dural sinuses, middle meningeal artery, and falx cerebri are pain-sensitive areas, the choroid plexus does not possess the necessary nerve endings to perceive pain.

Submit
144. Hypersensitive pneumonitis is classically a:

Explanation

Hypersensitive pneumonitis is classically an immune complex mediated hypersensitivity. This means that it is caused by the formation of immune complexes in the lungs, leading to an inflammatory response. Immune complexes are formed when antigens and antibodies bind together, and they can deposit in the lungs and trigger an immune response. This type of hypersensitivity reaction involves the activation of complement proteins and the recruitment of inflammatory cells, leading to tissue damage and inflammation in the lungs.

Submit
145. A 55 years old lady posted for hip replacement surgery. All of the following are known causes of DVT except:

Explanation

Infections can cause DVT, but the question asks for the known causes of DVT that are NOT associated with hip replacement surgery. Therefore, the correct answer is infections, as they are not directly related to the surgery itself.

Submit
146. True about infantile polycystic kidney disease include the following except:

Explanation

Infantile polycystic kidney disease is a genetic disorder characterized by the presence of renal cysts at birth, hepatic cysts, and periportal fibrosis. The disease is inherited in an autosomal recessive manner, not autosomal dominant. Therefore, the correct answer is autosomal dominant, as it is not true about infantile polycystic kidney disease.

Submit
147.  In a lady with bilateral superior temporal quadrantopia, galactorrhea, the most probable cause is:

Explanation

In a lady with bilateral superior temporal quadrantopia and galactorrhea, the most probable cause is a pituitary macroadenoma. Pituitary macroadenomas are large tumors that can compress the optic chiasm, leading to visual field defects such as quadrantopia. They can also cause hormonal imbalances, including hyperprolactinemia, which can result in galactorrhea. Craniopharyngiomas are another type of pituitary tumor but they typically present with different visual field defects and do not commonly cause galactorrhea. Sheehan's syndrome is a condition caused by postpartum pituitary necrosis and would not typically present with visual field defects. Pituitary hypophysitis is an inflammation of the pituitary gland and is not commonly associated with visual field defects or galactorrhea.

Submit
148. Regarding Parvovirus B-19, false statement is:

Explanation

The false statement regarding Parvovirus B-19 is "Can cross placenta in". Parvovirus B-19 is a DNA virus that can cause anemia, but it does not have the ability to cross the placenta.

Submit
149. Regarding purification of water all are true except:

Explanation

Clostridial infection does not indicate recent infection. Clostridial infections are caused by bacteria of the Clostridium genus, which can cause various types of infections including tetanus and botulism. These infections can occur due to the presence of the bacteria in the environment or through contaminated wounds, and they are not necessarily indicative of a recent infection.

Submit
150. LPL incorrect is:

Explanation

The correct answer is "Does not require C-II as cofactor." This means that LPL (lipoprotein lipase) does not need C-II (cofactor II) in order to function. C-II is a protein that activates LPL and allows it to break down triglycerides. However, in this case, LPL is able to function without the presence of C-II.

Submit
151. Trotters triad include

Explanation

Trotter's triad is a set of three symptoms that are commonly seen together in certain medical conditions. Deafness, mandibular neuralgia, and palatal palsy are the three symptoms included in Trotter's triad. However, seizures are not typically considered a part of Trotter's triad. Therefore, the correct answer is seizures.

Submit
152. Valproate causes all except:

Explanation

Valproate is a medication commonly used to treat epilepsy and bipolar disorder. It is known to cause various side effects, including weight gain and alopecia (hair loss). However, it does not cause liver damage. In fact, liver damage is a rare side effect of valproate and is more commonly associated with other anti-epileptic drugs. Therefore, the correct answer is liver.

Submit
153. Treatment for leukemia in a child with hyperleukocytosis is all except:

Explanation

In the treatment of leukemia in a child with hyperleukocytosis, all options are appropriate except for immediately starting chemotherapy. Hyperleukocytosis refers to an extremely high white blood cell count, which can lead to complications such as blood vessel blockage. Before starting chemotherapy, it is important to stabilize the child's condition by providing IV fluids to prevent dehydration, administering allopurinol to reduce the risk of tumor lysis syndrome, and alkalinizing the urine to prevent kidney damage. Once the child is stable, chemotherapy can be initiated to target and kill the leukemia cells.

Submit
154. Tickborne relapsing fever is caused by all except:

Explanation

Tickborne relapsing fever is caused by several species of bacteria belonging to the genus Borrelia. B. recurrentis is the causative agent of louse-borne relapsing fever, not tickborne relapsing fever. Therefore, it is not one of the bacteria responsible for causing tickborne relapsing fever.

Submit
155. Best contraception for lactating mothers:

Explanation

POPs (progestin-only pills) are considered the best contraception for lactating mothers. This is because they contain only progestin hormone, which does not affect milk production or quality. POPs are safe to use while breastfeeding and do not have any negative impact on the baby's health. They are also convenient as they do not require daily administration and can be taken at any time. Additionally, POPs have a high effectiveness rate in preventing pregnancy when taken correctly.

Submit
156. The graph below depicts the results of a red cell osmotic fragility test. The dashed curve represents which of the following?

Explanation

The dashed curve in the graph represents hereditary spherocytosis. Hereditary spherocytosis is a condition characterized by the presence of spherocytes, which are smaller and more dense red blood cells. In the osmotic fragility test, spherocytes are more prone to hemolysis when exposed to hypotonic solutions, resulting in a higher fragility curve. Therefore, the dashed curve in the graph indicates increased osmotic fragility, which is consistent with hereditary spherocytosis.

Submit
157. Which is raised in dysgerminoma? (DNB JUNE)

Explanation

LDH, or lactate dehydrogenase, is an enzyme that is commonly elevated in dysgerminoma, a type of germ cell tumor that arises from the ovaries. Dysgerminomas are known to produce high levels of LDH, which can be detected through blood tests. AFP (alpha-fetoprotein) and HCG (human chorionic gonadotropin) are tumor markers that are typically elevated in other types of germ cell tumors, such as yolk sac tumors and choriocarcinomas. CA-A 19-9 is a tumor marker that is associated with certain types of gastrointestinal cancers and is not relevant to dysgerminoma.

Submit
158. Spastic paraplegia is caused by all except:

Explanation

Spastic paraplegia is a condition characterized by stiffness and weakness in the legs. It is caused by various factors, including vitamin B12 deficiency, cervical spondylosis, and amyotrophic lateral sclerosis. However, lead poisoning is not a known cause of spastic paraplegia. Lead poisoning primarily affects the nervous system and can lead to symptoms such as developmental delays, cognitive impairment, and behavioral problems, but it is not associated with spastic paraplegia.

Submit
159. Paraneoplastic syndrome associated with RCC are all of the following except:

Explanation

Paraneoplastic syndromes are a group of rare disorders that occur in some cancer patients. They are caused by substances released by the tumor or by an immune response triggered by the tumor. In renal cell carcinoma (RCC), paraneoplastic syndromes commonly include polycythemia (excessive production of red blood cells), hypercalcemia (elevated levels of calcium in the blood), and malignant hypertension (severely high blood pressure). However, Cushing syndrome, which is characterized by excessive cortisol production, is not typically associated with RCC as a paraneoplastic syndrome.

Submit
160. Abrasion marks resemble:

Explanation

Abrasion marks resemble ant-bite marks because both can appear as small, red, itchy bumps on the skin. Ant bites can cause a localized reaction, similar to an abrasion, where the skin is irritated and may have a raised bump or redness. Therefore, the appearance of abrasion marks can be similar to ant-bite marks.

Submit
161. Vitamin A prophylaxis program is an exmple of:

Explanation

The Vitamin A prophylaxis program is an example of specific protection because it specifically targets the prevention of Vitamin A deficiency and its associated health problems. This program aims to provide individuals with adequate amounts of Vitamin A to protect against the development of conditions such as night blindness and xerophthalmia. Specific protection interventions are designed to target specific diseases or health risks and involve measures such as immunizations, screenings, and prophylactic treatments to prevent or minimize the impact of these specific health issues.

Submit
162. Which diseases are given benefit under Disability Act except (DNB JUNE.):

Explanation

The correct answer is mental retardation. Mental retardation is given benefit under the Disability Act, which provides support and protection for individuals with disabilities. Schizophrenia, bipolar disorder, and dementia are not specifically mentioned in the question, so it can be inferred that they are not given benefits under the Disability Act.

Submit
163. All are branches of splenic artery except:

Explanation

The right gastroepiploic artery is not a branch of the splenic artery. It is actually a branch of the gastroduodenal artery, which is a branch of the common hepatic artery. The splenic artery gives rise to branches such as the short gastric artery, hilar artery, and arteria pancreatica magna. Therefore, the correct answer is the right gastroepiploic artery.

Submit
164. Psedoclaudication seen due to:

Explanation

Lumbar canal stenosis is a condition characterized by narrowing of the spinal canal in the lower back. This narrowing can compress the nerves that travel through the spinal canal, leading to symptoms such as pain, weakness, and numbness in the lower back, buttocks, and legs. Pseudoclaudication refers to leg pain that is similar to the pain experienced during vascular claudication, but is actually caused by nerve compression rather than arterial insufficiency. Therefore, lumbar canal stenosis is the most likely cause of pseudoclaudication among the given options.

Submit
165. Action potential generated in axon hillock is due to:

Explanation

The action potential generated in the axon hillock is due to the presence of more ion channels. Ion channels are responsible for controlling the flow of ions in and out of the neuron, which is essential for generating and propagating the action potential. The axon hillock, located at the base of the neuron, contains a high density of ion channels, making it an important site for initiating the action potential. This high concentration of ion channels allows for a rapid and efficient generation of the action potential, which is necessary for the transmission of electrical signals along the neuron.

Submit
166. True for Chlamydia trachomatis:

Explanation

Chlamydia trachomatis is susceptible to penicillin. This means that penicillin can effectively treat infections caused by this bacteria. Other antibiotics such as azithromycin or doxycycline can also be used to treat Chlamydia trachomatis infections, but penicillin is one of the options. It is important to note that not all strains of Chlamydia trachomatis may be susceptible to penicillin, and the choice of antibiotic should be based on the specific strain and individual patient factors.

Submit
167. Involvement of sweat glands and hair follicies with granuloma around the hair follicles is seen in:

Explanation

Lichen Scrofulosum is a skin condition characterized by the formation of granulomas around the hair follicles. This involves the sweat glands and hair follicles, leading to the development of small, itchy papules or nodules on the skin. The condition is typically caused by a bacterial infection, most commonly Mycobacterium tuberculosis, and is often seen in individuals with a weakened immune system. Treatment usually involves antibiotics to target the underlying infection and topical creams to alleviate symptoms.

Submit
168. Vaginal delivery is allowed in all except:

Explanation

Monochorionic monoamniotic twins are not allowed to be delivered vaginally due to the high risk of cord entanglement and compression. In this type of pregnancy, the twins share the same placenta and amniotic sac, increasing the chances of complications during delivery. Vaginal delivery is possible in the other three scenarios mentioned.

Submit
169. All are traumatic asphyxia except:

Explanation

Traumatic asphyxia occurs when there is a sudden and intense compression of the chest, leading to a lack of oxygen supply to the body. In a railway accident, road traffic accident, and stampede, there is a high likelihood of individuals being subjected to severe compression forces, such as being crushed or trampled, which can cause traumatic asphyxia. However, accidental strangulation involves external compression of the neck, obstructing the airway and causing asphyxia. It is not considered traumatic asphyxia because the compression force is not applied to the chest directly.

Submit
170. Which of the following won't be present in right heart failure?

Explanation

Right heart failure is characterized by the inability of the right side of the heart to effectively pump blood. This can lead to congestion in the systemic venous circulation. Increased pulmonary capillary wedge pressure (PCWP) is a hallmark of left heart failure, not right heart failure. In right heart failure, the right side of the heart is affected, causing blood to back up into the systemic circulation. This can lead to symptoms such as pulsatile liver, increased jugular venous pressure (JVP), and a positive hepatojugular reflex. Therefore, increased PCWP would not be present in right heart failure.

Submit
171. Which is high-risk for laparoscopic surgery?

Explanation

Obesity is considered high-risk for laparoscopic surgery because it can increase the difficulty of performing the surgery and increase the risk of complications. Obese patients often have excess abdominal fat, which can make it more challenging to access and visualize the surgical site. Additionally, obesity is associated with a higher risk of cardiovascular and respiratory complications during surgery. Therefore, obese patients require careful preoperative evaluation and may need additional precautions during laparoscopic surgery to ensure their safety.

Submit
172. Late expanding stagoe in demography is:

Explanation

Late expanding stage in demography refers to a stage in the demographic transition model where both birth rates and death rates are low. This stage typically occurs in developed countries with high levels of education, healthcare, and economic development. In this stage, the population growth rate slows down and stabilizes. The given answer, "BR in low DR v. low," correctly identifies this stage by stating that the birth rate is low compared to the death rate.

Submit
173. Thoracic outlet syndrome is diagnosed by:

Explanation

Thoracic outlet syndrome is a condition characterized by compression of the nerves or blood vessels in the thoracic outlet area, which is located between the neck and shoulder. Clinical examination is the most common method used to diagnose this condition. During the examination, a healthcare professional will assess the patient's symptoms, medical history, and perform physical tests to evaluate the range of motion, strength, and sensation in the affected area. This allows the healthcare professional to identify any abnormalities or signs of compression, helping to confirm the diagnosis of thoracic outlet syndrome. X-ray, electromyography, and CT scan may be used as additional diagnostic tools to further evaluate the condition, but clinical examination is the primary method for diagnosis.

Submit
174. Pregnant women in 1st trimester is given spiramycin that she did not take complete treatment and baby is born with hydrocephalus. From which infection she was suffering?

Explanation

The pregnant woman was suffering from Toxoplasma infection. Spiramycin is a medication that is commonly used to treat Toxoplasma infection in pregnant women. If the woman did not complete the treatment, it is possible that the infection was not fully controlled, leading to complications such as hydrocephalus in the baby. HSV, Treponema pallidum, and CMV are other infections that can affect pregnant women, but in this case, the correct answer is Toxoplasma.

Submit
175. Least polar among following groups:

Explanation

The methyl group is the least polar among the given groups. This is because it consists of only carbon and hydrogen atoms, which have similar electronegativities. As a result, there is no significant difference in electron distribution, leading to a nonpolar nature. In contrast, the other groups (amino, carboxyl, and phosphate) contain atoms with higher electronegativities, such as nitrogen, oxygen, and phosphorus, which can create polar bonds and contribute to overall polarity.

Submit
176. False-positive rheumatoid factor can be associated with all except:

Explanation

False-positive rheumatoid factor can be associated with various conditions and infections, but it is not typically associated with inflammatory bowel disease. Rheumatoid factor is an antibody that is commonly found in individuals with rheumatoid arthritis, but it can also be present in other conditions such as hepatitis B infection (HbsAg), syphilis (VDRL), and autoimmune hemolytic anemia (Coombs test). However, there is no known association between false-positive rheumatoid factor and inflammatory bowel disease.

Submit
177. Renal calculi associated with Proteus infection is:

Explanation

Renal calculi associated with Proteus infection are typically composed of triple phosphate. Proteus bacteria produce urease, an enzyme that hydrolyzes urea to ammonia and carbon dioxide. The ammonia produced raises the urine pH, creating an alkaline environment that promotes the formation of triple phosphate crystals. These crystals can then aggregate and form renal calculi, also known as kidney stones. Therefore, triple phosphate is the correct answer for this question.

Submit
178. Entrapment neuropathies commonly affect the following nerves except:

Explanation

The question asks for a nerve that is not commonly affected by entrapment neuropathies. The correct answer is "Femoral." This means that entrapment neuropathies commonly affect the tibial, lateral cutaneous nerve of thigh, and median nerves, but not the femoral nerve.

Submit
179. Nitroglycerin causes all except:

Explanation

Nitroglycerin is a medication that is commonly used to treat angina and heart conditions. It works by relaxing and dilating the blood vessels, which helps to increase blood flow and reduce the workload on the heart. As a result, it causes vasodilation, which is the widening of blood vessels. This leads to a decrease in blood pressure, known as hypotension. However, it does not cause bradycardia, which is a slow heart rate. Methemoglobinemia, a condition where the blood is unable to carry oxygen effectively, can be a side effect of nitroglycerin use. Therefore, the correct answer is "Hypotension and bradycardia."

Submit
180. All are compoite muscles except:

Explanation

The correct answer is Flexor carpi ulnaris. This muscle is not a composite muscle, meaning it is not composed of multiple muscle heads or parts. Instead, it is a single muscle with a single origin and insertion. In contrast, the other three muscles listed (Pectineus, Biceps femoris, and Flexor digitorum profundus) are composite muscles, meaning they are made up of multiple muscle heads or parts that have different origins and/or insertions.

Submit
181. Vaccine with best efficacy:

Explanation

Measles vaccine has the best efficacy among the given options. Measles is a highly contagious viral disease that can lead to severe complications and even death. The measles vaccine is highly effective in preventing measles infection, with a high level of protection after two doses. It has been proven to be safe and has significantly reduced the incidence of measles worldwide. The vaccine has a high efficacy rate in preventing measles and its associated complications, making it the best choice among the given options.

Submit
182. SIRS includes all except:

Explanation

SIRS, or Systemic Inflammatory Response Syndrome, is a condition characterized by a widespread inflammatory response in the body. It is typically associated with an infection or injury. Leukocytosis, hypothermia, and an oral temperature of more than 38°C are all common manifestations of SIRS. Thrombocytopenia, on the other hand, refers to a low platelet count in the blood and is not typically seen in SIRS. Therefore, thrombocytopenia is the exception among the given options.

Submit
183. Enterovirus causes all except:

Explanation

Enterovirus is a type of virus that commonly causes aseptic meningitis, pleurodynia, and herpangina. Aseptic meningitis is an inflammation of the membranes surrounding the brain and spinal cord. Pleurodynia is a condition characterized by sudden sharp pain in the chest and abdomen. Herpangina is a viral infection that causes painful sores in the throat and mouth. However, enterovirus does not typically cause hemorrhagic fever, which is a severe condition characterized by bleeding and organ failure.

Submit
184. True about leptospirosis is all except:

Explanation

Leptospirosis is a bacterial infection that is typically transmitted through contact with water or soil contaminated with the urine of infected animals, especially rodents. It can cause severe symptoms and, in severe cases, penicillin given intravenously is the recommended treatment. Additionally, the mortality rate in severe cases of leptospirosis is estimated to be between 5 to 15%. However, the correct answer is that antibodies can be detected in a week, which is not true. It usually takes around 2 to 4 weeks for antibodies to be detectable in the blood after infection.

Submit
185. All are true about pre-eclampsia except:

Explanation

Pre-eclampsia is a condition that occurs during pregnancy, characterized by high blood pressure and damage to organs such as the liver and kidneys. It can lead to complications such as pulmonary edema, acute renal failure (ARF), and deep vein thrombosis (DVT). However, cerebral hemorrhage, which refers to bleeding in the brain, is not typically associated with pre-eclampsia. Therefore, the correct answer is cerebral hemorrhage.

Submit
186. MICRO RNA function:

Explanation

Micro RNA (miRNA) plays a crucial role in gene regulation. It functions by binding to target messenger RNA (mRNA) molecules and inhibiting their translation into proteins. By doing so, miRNA can control the expression of genes and regulate various cellular processes. This regulation is essential for maintaining normal cell function and development. Therefore, the correct answer is "Gene regulation."

Submit
187. Not a treatment for chronic backache:

Explanation

Epidural steroid injections are commonly used to provide temporary relief from chronic back pain by reducing inflammation in the affected area. However, they are not considered a long-term treatment for chronic backache. Instead, exercises are often recommended to strengthen the muscles supporting the spine and improve flexibility. Nonsteroidal anti-inflammatory drugs (NSAIDs) can also help reduce pain and inflammation, while bed rest for three months is generally not recommended as it can lead to muscle weakness and stiffness.

Submit
188. In fetus Angio genesis in eye involve all except:

Explanation

In the fetus, angiogenesis in the eye is a process that involves the growth of new blood vessels. TNF, BFGF, and VEGF are all known to play important roles in angiogenesis. However, IL-8 is not typically associated with angiogenesis in the eye during fetal development. Therefore, IL-8 is the correct answer as it is the exception among the given options.

Submit
189. Most associated with moderate risk of breast carcinoma:

Explanation

Atypical hyperplasia is most associated with a moderate risk of breast carcinoma. This condition involves abnormal growth of breast cells, which can increase the risk of developing breast cancer. Atypical adenosis, metaplasia, and atypical hypertrophy are also associated with an increased risk, but atypical hyperplasia is considered to have the highest risk among these options.

Submit
190. All are the contents of depp perineal pouch except:

Explanation

The contents of the deep perineal pouch include the dorsal nerve of the penis, the sphincter urethra, and the bulbourethral glands. However, the bulb/root of the penis is not a content of the deep perineal pouch.

Submit
191. Neural tube begin to close from which region

Explanation

The neural tube begins to close from the cervical region. The neural tube is a structure that eventually develops into the brain and spinal cord during embryonic development. Closure of the neural tube is a critical step in the formation of the central nervous system. It starts at the cervical region, which is the area of the spine located in the neck.

Submit
192. All lead to hyper coagulability except:

Explanation

Surgery of 1 hour duration does not lead to hypercoagulability. While burns, hemorrhage, and infections can all trigger a hypercoagulable state, a short surgery duration does not typically have the same effect.

Submit
193. All of the following is feature of temporal lobe epilepsy except:

Explanation

Temporal lobe epilepsy is characterized by seizures originating in the temporal lobe of the brain. Atrophy, or shrinkage, of the temporal lobe is a common finding in temporal lobe epilepsy. Additionally, atrophy of the hippocampus and fornices (a bundle of nerve fibers) is also commonly observed. However, atrophy of the mammillary body is not typically associated with temporal lobe epilepsy. Therefore, the correct answer is "Atrophy of mammillary body."

Submit
194. Restless leg syndrome (RLS) is seenin:

Explanation

Restless leg syndrome (RLS) is a neurological disorder characterized by an irresistible urge to move the legs, often accompanied by uncomfortable sensations. It is commonly associated with chronic renal failure, a condition in which the kidneys are unable to adequately filter waste products from the blood. This is because kidney dysfunction can lead to imbalances in electrolytes and minerals, such as calcium, phosphorus, and potassium, which can contribute to the development of RLS symptoms. Therefore, chronic renal failure is a known risk factor for RLS.

Submit
195. Scarring alopecia is associated with:

Explanation

Scarring alopecia refers to a type of hair loss that is caused by the destruction of hair follicles, leading to permanent hair loss and scarring on the scalp. Lichen planus is a chronic inflammatory skin condition that can affect the scalp and cause scarring alopecia. It is characterized by itchy, flat-topped, purple-colored bumps on the skin. Other conditions listed in the options, such as alopecia areata, tinea capitis, and androgenic alopecia, do not typically cause scarring and are associated with different forms of hair loss.

Submit
196. A five-year-old boy has precocious puberty bp 130/80. Estimation of which of the following will help in diagnosis?

Explanation

Precocious puberty refers to the early onset of puberty before the age of 8 in girls and 9 in boys. It is often caused by an excess production of adrenal androgens. 11-Deoxycortisol is a precursor hormone in the production of cortisol and androgens. Elevated levels of 11-Deoxycortisol can indicate an adrenal disorder such as congenital adrenal hyperplasia (CAH), which can cause precocious puberty. Therefore, estimation of 11-Deoxycortisol can help in the diagnosis of the underlying cause of precocious puberty in this five-year-old boy.

Submit
197. About VHL which is not true:

Explanation

The given answer states that gastric carcinoma is not true about VHL (Von Hippel-Lindau disease). This means that gastric carcinoma is not a characteristic feature or manifestation of VHL. VHL is a genetic disorder that primarily affects the eyes, brain, and other parts of the body. It is characterized by the development of retinal and cerebellar hemangioblastomas (tumors), pheochromocytomas (tumors of the adrenal glands), and renal cell carcinoma (kidney cancer). However, gastric carcinoma is not typically associated with VHL.

Submit
198. A male child presents with delayed development and biting of the lips and hands. His parent have restrained him because he obsessively chews on his lips and fingers which of the following is likely to occur in this child?

Explanation

The correct answer is HGPRTase def. This is likely to occur in the child because the symptoms mentioned, delayed development and biting of the lips and hands, are characteristic of Lesch-Nyhan syndrome, which is caused by a deficiency of the enzyme hypoxanthine-guanine phosphoribosyltransferase (HGPRTase). This enzyme is involved in the salvage pathway of purine metabolism, and its deficiency leads to the accumulation of uric acid, neurologic abnormalities, and self-injurious behaviors such as lip and finger biting.

Submit
199. Carpal tunnel syndrome can be caused by the following except:

Explanation

Addison's disease is a condition that occurs when the adrenal glands do not produce enough hormones, particularly cortisol and aldosterone. Carpal tunnel syndrome is a condition that occurs when there is pressure on the median nerve in the wrist. While diabetes mellitus, acromegaly, and hypothyroidism have all been associated with an increased risk of carpal tunnel syndrome, there is no known link between Addison's disease and carpal tunnel syndrome. Therefore, Addison's disease is the exception among the given options.

Submit
200. True about temporal arteritis all except:

Explanation

Temporal arteritis, also known as giant cell arteritis, is a condition characterized by inflammation of the blood vessels, especially the temporal arteries. It is more common in females and is typically seen in elderly women. The inflammation can lead to a variety of symptoms, including headache, jaw pain, and vision problems. However, exposure to heat does not worsen the condition. Therefore, the statement "Worsens on exposure to heat" is not true about temporal arteritis.

Submit
View My Results

Quiz Review Timeline (Updated): Nov 25, 2024 +

Our quizzes are rigorously reviewed, monitored and continuously updated by our expert board to maintain accuracy, relevance, and timeliness.

  • Current Version
  • Nov 25, 2024
    Quiz Edited by
    ProProfs Editorial Team
  • Jun 15, 2014
    Quiz Created by
    Fmgs India
Cancel
  • All
    All (200)
  • Unanswered
    Unanswered ()
  • Answered
    Answered ()
Karyotyping is done for:
Artery in anatomical snuff box:
The receptor of HIV:
Which of the following is protective against carcinoma colon?
Neurofibromatosis is associated with all except:
A person sits from standing position, change seen is:
A 34-year-old female presents with symptoms of cough, dyspnea,...
Sleep is primarily regulated by:
After cutting DNA with restriction enzymes; segments are joined...
Q fever is transmitted by:
Laryngeal pseudosulcus true is:
 Gold standard to diagnosis insulinoma is:
Staging for Wilms syndrome:
Prunning of vessels is seen in:
Chief cells are found in:
In HIV maximum risk of transmission is by:
Characteristic of acute inflammation:
Hallmark feature of benign HTN is:
Which of the following is the recommended timing for the...
An intrinsic factor of the castle is present in:
Extensive abrasions are found on the body of a pedestrian lying...
Amifostine is protective to all except:
A 7-year-old girl presents with bleeding in joints. She has...
Faciliatated diffusion:
The modes of infection of the following diseases are all except:
True for hemochromatosis:
What is intermittent claudication?
Patient with foreign body sensation in eye and swollen knee it...
Which is not involved in the intrinsic pathway?
A Down syndrome patient is posted for surgery. The necessary...
Marker of neural tube defect:
Incidence rate can be calculated by:
Most recent classification according to WHO for disability:
A man with blunt injury abdomen after road side accident has BP...
Central chemoreceptors are most sensitive to:
Diastolic pressure in aorta is maintained by:
Sertoli cells are associated with:
All drugs can be given to a mother with lupus who isn on 35th...
True for necrotizing fasciitis are all except:
Casper's dictum is for:
Cut off of Indian reference man (kg):
STEPS refers to:
Most important prognostic factor of colorectal carcinoma:
Neurofibromatosis is associated with all except:
True statement about RDA:
What is the most common cause of death after total hip...
Pt. presents with punched out lesion on X-ray withSerum Na-144, K+,...
Spongy part of the male urethra drains via which lymph nodes:
Nutrition in the community is assessed by all/except:
A young man with gout has synovial fluid removed. It would shows:
The following drug can be given safely in pregnancy:
Urogenital diaphragm is contributed by all except:
Organelle which plays a pivotal role in apoptosis:
A girl-aged 8 years has been admitted for dialysis. She ahs serum...
Bleomycin toxicity affects which type of cells:
Difference between follicular adenoma and carcinoma:
Positive hepatojugular reflux is found in conditions except:
A neonate with focal skin lesions and hypoplastic limbs causative...
A 29 years old person known diabetic on OHA since 3 years. He has...
Angiotensin II causes all except:
Disease, which permanently alters finger print: (DNB JUNE)
Patient with pancreatic transplant with urinary drainage....
Acetyl CoA can be directly converted to all except:
In surgical staging of ovarian Ca, all are done except:
Best marker for stratification of Carciovascular disease risk:
C wave in JVP indicates:
Mifepristone is used in the management of:
The following are the complications of the HIV infection except:
About renal physiology all are true except:
Oxygen demand of heart:
In simple random sampling:
True regarding DMPA include the following except:
Characteristic of HbS is all except:
Finnish type of nephritic syndrome is associated with:
Ca breast is not discovered in mammography in young woman due to:
Ifosfamide is:
Which structure is just lateral to anterior perforated substance?
Causes of the sigmoid shape of the oxygen-hemoglobin dissociation...
Test for assessment of the mucosal function of GIT:
If a patient with severe hyperglycemia is given IV insulin, which...
A young man met with an accident leading to loss of hearing in...
All of the following arteries supply medulla except:
Which structure is just lateral to anterior perforated substance?
Superior gluteal nerve supplies all except:
Appetite suppressors are all except:
In obstructive azoospermia:
Minimum duration between onset of symptoms and death is seen in:
Maltese cross is associated with:
Heat hematoma differs from blunt trauma by:
Splicing is associated with:
In a standard distribution curve:
What is specific for GIST?
When one gene is inherited from one parent only, it is known as:
What is affected in HbS (Hemoglobin S)?
All is true for Sternberg canal except:
Coagulative necrosis is found in which infection?
Immediately after eating a man developed lump in throat,...
All are the side effects of tacrolimus except:
A most common sign of acute hypoxia in neonates:
Which of the following is not true about JRA?
Following is the NCCT head of a 40-year-old patient who is...
What is not true about nontyphoidal Salmonella?
After 48 hours of fasting insulin receptors are down regulated...
CAP in Lac Operon is an example of:
Use of tamoxifen in HRT does not lead to
A patients is given tacrolimus, which antibiotic should not be...
Cell-cell interaction occurs by all of the following mechanisms...
The status of fluid in distal convoluted tubule is all except:
True about deglutition persistalsis of esophagus:
Potential for Reperfusion of the tissue, post-infarct can be...
Best material for below inguinal arterial graft:
The patient has chronic arterial obstruction. He presents with:
Coombs positive hemolytic anemia associated with:
About Hepatitis C, which is true?
Which of these is an FDA approved indication for use of modafinil...
Triple assessment for Ca breast is:
Primary Skin lesions are seen in all except:
In a program the end point of all activities which may not be...
Myodesis is contraindicated in:
Meningitis spread from CNS to Inner ear through:
Adrenal adenoma on CT which is not true:
Aldosterone leads to all except:
Posterior superior alveolar nerve is a branch of:
GALT (Gut Associated Lymphoid Tissue) is present in:
Paravertebral black, can extend into all except:
E.coli 0157 is cultured not on:
Pasturella multocida infection is due to:
MC cause of superficial thrombophlebitis:
False about Aedes aegypti is:
Baby friendly hospital initiative all except:
Popliteal pulse is not felt clearly because:
Regarding angioneurotic edema, what is not true?
Dengue hemorrhagic fever occurs in:
The creamy fishy odor is caused by:
Non-invasive diarrhea caused by all except:
Muscle's blood supply increases during exercise due to:
Energy expenditure in resting state depends upon:
Regarding crude birth rate all are true except:
Primary amenorrhea is not seen in:
All is true about trochlear nerve except:
Calcification of posterior spinal ligament is best diagnosed by:
Psammoma bodies are seen in all except:
All are pain sensitive area of brain except:
Hypersensitive pneumonitis is classically a:
A 55 years old lady posted for hip replacement surgery. All of...
True about infantile polycystic kidney disease include the...
 In a lady with bilateral superior temporal quadrantopia,...
Regarding Parvovirus B-19, false statement is:
Regarding purification of water all are true except:
LPL incorrect is:
Trotters triad include
Valproate causes all except:
Treatment for leukemia in a child with hyperleukocytosis is all...
Tickborne relapsing fever is caused by all except:
Best contraception for lactating mothers:
The graph below depicts the results of a red cell osmotic...
Which is raised in dysgerminoma? (DNB JUNE)
Spastic paraplegia is caused by all except:
Paraneoplastic syndrome associated with RCC are all of the...
Abrasion marks resemble:
Vitamin A prophylaxis program is an exmple of:
Which diseases are given benefit under Disability Act except (DNB...
All are branches of splenic artery except:
Psedoclaudication seen due to:
Action potential generated in axon hillock is due to:
True for Chlamydia trachomatis:
Involvement of sweat glands and hair follicies with granuloma...
Vaginal delivery is allowed in all except:
All are traumatic asphyxia except:
Which of the following won't be present in right heart failure?
Which is high-risk for laparoscopic surgery?
Late expanding stagoe in demography is:
Thoracic outlet syndrome is diagnosed by:
Pregnant women in 1st trimester is given spiramycin that she did...
Least polar among following groups:
False-positive rheumatoid factor can be associated with all...
Renal calculi associated with Proteus infection is:
Entrapment neuropathies commonly affect the following nerves...
Nitroglycerin causes all except:
All are compoite muscles except:
Vaccine with best efficacy:
SIRS includes all except:
Enterovirus causes all except:
True about leptospirosis is all except:
All are true about pre-eclampsia except:
MICRO RNA function:
Not a treatment for chronic backache:
In fetus Angio genesis in eye involve all except:
Most associated with moderate risk of breast carcinoma:
All are the contents of depp perineal pouch except:
Neural tube begin to close from which region
All lead to hyper coagulability except:
All of the following is feature of temporal lobe epilepsy except:
Restless leg syndrome (RLS) is seenin:
Scarring alopecia is associated with:
A five-year-old boy has precocious puberty bp 130/80. Estimation...
About VHL which is not true:
A male child presents with delayed development and biting of the...
Carpal tunnel syndrome can be caused by the following except:
True about temporal arteritis all except:
Alert!

Advertisement